If the probability that a person will die in the next year is 771/100,000
what is the probability that the person will not die in the next year?

A. 0.771%
B. 99.229%
C. 0.229%
D. 99.771%

Answers

Answer 1

Answer:

(B) 99.229%

Step-by-step explanation:

If the probability of someone dying next year is [tex]\frac{771}{100000}[/tex], that means the probability of someone NOT dying next year is [tex]\frac{100000-771}{100000} = \frac{99229}{100000}[/tex].

To find the percent value of this fraction, we divide 99229 by 100000 and multiply by 100.

[tex]\frac{99229}{100000} = 0.99229\cdot100=99.229[/tex]

So, the answer is B, 99.229%.

Hope this helped!

Answer 2
The answer to this question is B. Or 99.229%

Related Questions

Someone pls help . Thank you sm ! i forgot how to do this lol .

Answers

Answer:

x = - 20, h = 34

Step-by-step explanation:

i can't tell ypou what the class period is, but for -x = 20, multiply each side by -1, then you get x = -20

-32 = 2-h

+32 .  +32

0=34-h

+h .   +h

34=h

flip

h=34

X= -20 and H= 34 the first one you just carry the negative sign to the 20

Which of the following represents continuous data? A. Amount of coins in your pocket B. Time it takes to count the coins in your pocket C. Number of collector coins you own D. Value of the coins in your pocket

Answers

Answer:

Time it takes to count the coins in your pocket

Step-by-step explanation:

answer is b. time it takes to count coins in your pocket

At the same time, Sam and Fred who are 168 miles apart begin moving toward each other. Find the number of hours before they meet if Sam runs 12 mph and Fred runs 9 mph.

Answers

Answer: 8 hours

Step-by-step explanation:

Let x be the time before they meet ( in hours)

Speed of Sam = 12 mph

Sped of Fred = 9 mph

Distance = Speed × time

Distance covered by Sam in x hours = 12x

Distance covered by Fred in x hours = 9x

                                               (Distance)     (rate1) (time1)          (rate2) (time2)  

Total distance between them: 168             =     12      x        +      9    x

[tex]\Rightarrow\ 168= 21x\\\\\Rightarrow\ x=\dfrac{168}{21}=8[/tex]

Hence,  the number of hours before they meet if Sam runs 12 mph and Fred runs 9 mph. = 8

The answer is 8 hourss.

Describe the transformations that will map triangle A to triangle B and illustrate the similarity between the two triangles.


A) reflect triangle a across the y-axis and then translate triangle a 5 units down.  B) reflect triangle A across the x-axis and then reflect triangle a across the y-axis.  
C) translate triangle a 4 units down and then rotate triangle A 180° counterclockwise about the origin. 
D) reflect triangle a across the x-axis and then rotate triangle A 90° counterclockwise about the origin

Answers

Honestly I’m not for sure but I’m think it’s either a or b

The triangle is shifted 4 units downwards and then is rotated about the origin.

What is image rotation?A rotation is a type of transformation which is a turn.A figure can be turned clockwise or counterclockwise on the coordinate plane.In both transformations the size 3 shape of the figure stays exactly the same.

Given are triangles A and B.

Transformation 1 : The triangle is shifted 4 units downwards.

Transformation 2 : The figure is rotated about the origin.

Therefore, the triangle is shifted 4 units downwards and then is rotated about the origin.

To solve more questions on dilation, visit the link below -

brainly.com/question/2859427

#SPJ5

The following frequency table shows the number of hours each of the staff members at Eroy's Electronics slept Thanksgiving night. Find the mean number of hours of sleep.

Answers

Answer:

Mean

=Sigma (Number of sleep ×Number of employees) / Sigma Number of employees

= (3×1) + (4×0) + (5×0)+ (6×2)+(7×1)+(8×1) / 1+0+0+2+1+1

= 3+0+0+12+7+8/ 1+0+0+2+1+1

= 30 / 5

= 6

So the mean of number of hour sleep is 6 hours.

I hope this helps

if u have question let me know in comments ^_^

Answer:

Step-by-step explanation:

Mean

=Sigma (Number of sleep ×Number of employees) / Sigma Number of employees

= (3×1) + (4×0) + (5×0)+ (6×2)+(7×1)+(8×1) / 1+0+0+2+1+1

= 3+0+0+12+7+8/ 1+0+0+2+1+1

= 30 / 5

= 6

So the mean of number of hour sleep is 6 hours.

I hope this helps

How would I solve this question? y = -1 1/8 - 7/8x -4x + 9y = -22 x = ?, y = ?

Answers

Is this two separate equations?
i think y is 7.75 or 7 3/4

During a certain 25-year period, the consumer price index (CPI) increased by 99%,but during the
next 25-year period, it increased by only 1%. Which of these conditions must have existed during the
second 25-year period?
A. Conflation
B. Deflation
C. Inflation
D. Stagnation​

Answers

I think the answer to this is D or stagnation

For the next 25-year period,(CPI) is increased by only 1% due to Inflation and Stagnation.

What is Stagnation ?

Stagnation is a situation of slow economic growth and relatively high unemployment (which basically means that aggregate production is reducing and some of the inputs of the economy, such capital or labor, are unemployed) , accompanied by rising prices, or inflation.

In terms of national accounting, it means a reduction in gross domestic product (GDP), with inflation (rise in all prices in the economy).

In terms of aggregate demand and supply models, stagnation is the result of a contraction of aggregate supply, which ceteris paribus, results in lower levels of production and higher prices.

Many theories have tried to explain this phenomena. Common interpretations link stagnation with and increase of the cost of production in the economy (that might be generated by an increase of gasoline for example), and its implications in the production (lower production because of higher costs), consequently unemployment and a rise of prices due to the increase of cost.

Here, During a certain 25-year period, the consumer price index (CPI) increased by 99% and during the next 25-year period, it increased by only 1% which shows slow economic growth which can be due to Inflation and Stagnation.

​Learn more about "Inflation" here:

https://brainly.com/question/28190771

#SPJ2

Find the volume. Show work plz

Answers

Answer:

[tex]93\frac{1}{3}[/tex] [tex]yd^{3}[/tex]

Step-by-step explanation:

The volume of a pyramid is:

[tex]V=\frac{lwh}{3}[/tex] (basically saying that you multiply the length, width, and the height together and then you divide that by 3)

Now, let's do the first part.

8 × 5 × 7 = 280

Now, let's divide.

280 ÷ 3 = 93 1/3

That's your answer!

Hope that helps and maybe earns a brainliest!

Have a great day! :)

Answer: 140 yards cubed

Step by step: formula- height•width•length/2
8•5=40
40•7=280
280/2=140
140 yards cubed

Patrick graphed (shown below) the high temperatures (in °C) that he experienced on each day of his trip to
Homer, Alaska
What is the meaning of point A?​

Answers

Answer:

(7, -5.5)

Step-by-step explanation:

is seen below on this graph (7, -5.5) shows x = 7 and y = -5.5

(7, 5.5) look ^^ they should help u

7/13 + 1/5 (its in fractions)

Answers

Answer:

48 ÷ 65

Step-by-step explanation:

hope it helps you

Answer:

[tex]\frac{48}{65}[/tex]

Step-by-step explanation:

Step 1: Find LCM for denominators

13(5) = 65

So we need to convert our fractions to add them

Step 2: Convert [tex]\frac{7}{13}[/tex] by multiplying both top-bottom by 5

[tex]\frac{7}{13} (\frac{5}{5} ) = \frac{35}{65}[/tex]

Step 3: Convert [tex]\frac{1}{5}[/tex] by multiplying both top-bottom by 13

[tex]\frac{1}{5} (\frac{13}{13} ) = \frac{13}{65}[/tex]

Step 4: Add the converted fractions

[tex]\frac{13}{65} +\frac{35}{65} =\frac{48}{65}[/tex]

And we have our answer!

Find the value of y and x.

Answers

Answer:

x = 130°y = 65°

Step-by-step explanation:

All inscribed angles intercepting the same arc have the same measure.

If you slide the vertex of angle y clockwise around the circle until it coincides with the vertex of the angle marked 65°, you see that those two inscribed angles (y and 65°) intercept the same arc, so have the same measure:

  y = 65°.

The measure of an inscribed angle is half the measure of the intercepted arc, so the arc marked x is double the angle 65°.

  x = 130°.

The answer for this is y= 65 degrees and x= 130 degrees

6.21(3.1) answer for number 6

Answers

The answer is 19.251

Someone pls help me . Thank you sm

Answers

━━━━━━━☆☆━━━━━━━

▹ Answer

X = 24

m = 13/7 or about 1.86

▹ Step-by-Step Explanation

X/3 + 7 = 15

X + 3 * 7/3 = 15

X + 21/3 = 15

X = 24

3.5m - 1.5 = 5

35/10m - 15/10 = 5

5 * 7/2 * 5m - 3 * 5/2 * 5 = 5

m = 13/7 or about 1.86

Hope this helps!

CloutAnswers ❁

━━━━━━━☆☆━━━━━━━

answer: x=24


explanation:

What is the area of the polygon shown below? (in the image). A. 322 mm^2 B. 364 mm^2 C. 520 mm^2 D. 584 mm^2 Show all work please!

Answers

Answer:

A.

Step-by-step explanation:

Find the area of the triangle: 26-20=6. 14x6=84/2=42. Now that we calculated the triangle's area, we need to find out the rectangle's. 20x14=280. Add them up: 280+42=322 mm^2. Triangle: b x h/2. Rectangle: l x h.

The area of a 2D form is the amount of space within its perimeter. The area of the given figure is 322 mm². The correct option is A.

What is an area?

The area of a 2D form is the amount of space within its perimeter. It is measured in square units such as cm², m², and so on. To find the area of a square formula or another quadrilateral, multiply its length by its width.

The given figure can be broken into a rectangle and a right-angle triangle, in order to calculate the area of the given polygon.

The area of the rectangle with a length of 20mm and height of 14 mm is,

Area of the rectangle = 20mm × 14mm

                                    = 280 mm²

The area of the right-angled triangle with a base length of 6mm and height of 14 mm is,

Area of the triangle = (1/2) × 6mm × 14mm

                                 = 42 mm²

Now, the total area of the given figure can be written as,

The total area of the given figure

= Area of the rectangle + Area of the triangle

= 280 mm² + 42 mm²

= 322 mm²

Hence, the area of the given figure is 322 mm².

Learn more about the Area here:

https://brainly.com/question/1631786

#SPJ5

you move up 7 units and left 6 units. you end at (-1,5). where did you start?​

Answers

Answer:I beleive it is (5,-2)

Step-by-step explanation:

First you wanna go backwards so start from (-1,5) and go down 7 units and 6 units right then you’ll get where you started ❤️ hope this helped

What is the slope of the line through (2,-2) and (9, 3)

Answers

Answer:it is 5/7

Step-by-step explanation: the formula for slope is rise/run divide and you get 5/7

Answer:

slope = [tex]\frac{5}{7}[/tex]

Step-by-step explanation:

Calculate the slope m using the slope formula

m = [tex]\frac{y_{2}-y_{1} }{x_{2}-x_{1} }[/tex]

with (x₁, y₁ ) = (2, - 2) and (x₂, y₂ ) = (9, 3)

m = [tex]\frac{3+2}{9-2}[/tex] = [tex]\frac{5}{7}[/tex]

Town B is 250 km from town A on a bearing of 080°.
Town C is 250 km from town B on a bearing of 220°.
What is the bearing from town A to town C?

Answers

Answer:

Step-by-step explanation:

220-80
The answer would be 140

Please help with 2 and 3 for maths

Answers

Answer:2-4=

Step-by-step explanation:y-16-(-42)

y-16+16

y-32

y-16-(-22)

y-16+4

y-20

y-16-(02)

y-16

y-16-(42)

y-0

Answer:

The answer is 2-4

Step-by-step explanation:

Please help ASAP!!!!!!!

Answers

Answer:

  $86.81

Step-by-step explanation:

Using the given formula, we want to compute A for ...

  P = 4750

  r = 0.2279

  n = 365 . . . . . assuming "exact" interest

  t = 1 or 30

For 1 day late:

  A = 4750(1 +0.2279/365)^(365·(1/365)) = 4752.97

For 30 days late:

  A = 4750(1 +0.2279/365)^(365·(30/365)) = 4839.78

The difference in these payment amounts is ...

  $4839.78 -4752.97 = $86.81

You would save $86.81 in interest charges by paying only 1 day late.

_____

Comment on the question

It would be a poor choice of credit card to use one that compounds interest daily. Most do so on a monthly basis.

Answer 86.61 !


Explanation : copied other person if you don’t mind

A vending machine accepted any combination of nickels, dimes, and quarters that added to $0.40. How many different combinations of coins were possible?

Answers

Answer: 1 quarter, 1 dime and 1 nickel.

Step-by-step explanation:

Quarter: 25 cents.

Dime: 10 cents

Nickel: 5 cents

25 + 10 = 35

35 + 5 = 40

Answer: 25 (1 quarter), 10 (1 dime), 5 (1 nickel) is 0.40 cents when all three numbers are summed up.

25 for one quarter and the other ones 5 cents and 10 cents

HELP MEH HELP MEH PLZ

Answers

Answer:

[2×10*(-3) × (4×10*(4)] = 80

Step-by-step explanation:

First of all 0.002 is 10*(-3) and not 10(-2) , so you can eliminate the first 2 answers.

Second of all, according to the calculator the left equation is correct, while the right one isn't... so there you go

Glad to help :)

The answer is the 3rd one you can eliminate the first two because they would be incorrect but if you want to check you answer you can enter the 3rd one on a scientific calculator

I NEED THE ANSWER ASAP

The number of students who smoke cigarettes at Broxton College is decreasing at a rate of one smoker every 6.31 days. At what rate in smokers per year is the number of smokers declining? Assume 365 days in a year and round to the nearest tenth of a smoker per year

Answers

Answer:

57.8

Step-by-step explanation:

We can set up a proportion for this, assuming x is the smokers lost in 365 days.

[tex]\frac{1}{6.31} = \frac{x}{365}[/tex]

Using the cross products property, we know that x will be equal to:

[tex](365\cdot1) \div 6.31\\365\div6.31\\\\\approx 57.8[/tex]

Hope this helped!

57.8 is the answer to this problem

kristen and melissa spent 35% of their $32 on movie tickets. how much money did they spend

Answers

Answer:

11.2$

Step-by-step explanation:

Kristina and Melissa had 32$ at total

● 32$ => 100%

They have spent 35%

Let x be that amount

● x => 35%

●32 => 100

● x => 35

● x = (35×32)/100 = 11.2$

They have spent 11.2$

Answer:

They spent $20.80

Step-by-step explanation:

Since they had $32 and spent 35% of it you would do 32 * 35%

 then you would get 11.2

now that is not the answer because that is just what 35% of 32 is

to get the answer you would then subtract 11.2 from 32 to get 20.8

and since this is a matter of money you would write 20.8 as $20.80.

For lunch, you get tacos that cost $5.50. You also get chips and salsa for $3.99. What is the total cost of your lunch?

Answers

Answer:

9.49

Step-by-step explanation:

Add the price of the chips and salsa to the price of the tacos

5.50

+3.99

---------------

9.49

Answer: $9.49

Step-by-step explanation:

0+9 is 9 and 5+9 is 14 but carry the 1 to the left and add 5+3+1 and you get your answer $9.49

$5.50

+$3.99

————

$9.49

2. Solvex2 - 6x = -5 by completing the square , Show all work for the steps below . ( a ) Forx ? - 6x + = -5+ , what value of c is used to complete the square ? ( b ) Substitute the value for c in Part 2 ( a ) . Then complete the square to rewrite the equation as the square of a binomial . ( c ) Solve for x

Answers

Answer:

see explanation

Step-by-step explanation:

Given

x² - 6x = - 5

To complete the square

add ( half the coefficient of the x- term )² to both sides

x² + 2(- 3)x + 9 = - 5 + 9 [ part a, c = 9 ]

(x - 3)² = 4 [ part b ]

Take the square root of both sides

x - 3 = ± [tex]\sqrt{4}[/tex] = ± 2 ( add 3 to both sides )

x = 3 ± 2

Thus part c is

x = 3 - 2 = 1

x = 3 + 2 = 5

On a certain map, 3 inches represents 15 miles. Briarwood and Middletown are 5 inches apart on the map. What is the actual distance between Briarwood and Middletown? A. 25 mi B. 30 mi C. 50 mi D. 75 mi

Answers

Answer:

i think the answer is a. 25

There, hope it helps :)

What will I get for this 16 + x = 12

Answers

First step: subtract 16 from both sides to get x=12-16

Second step: simplify to x= -4

Third step: fill it back into the equation to check, 16 + -4 = 12, and 12=12 this is a true statement

Answer:

x = -4

Step-by-step explanation:

16 + x = 12

16 - 16 + x = 12 - 16

x = -4

PLEASE HELP WILL GIVE BRAINLY!!!!!!!!!What is the vertex of the graph of the function f(x) = x2 + 8x − 2 ?
(−4, 18)
(0, -2)
(-8, -2)
(−4, −18)

Answers

Answer: D.  (-4, -18)

===================================================

Explanation:

A graph is a nice addition (and it will likely help us see the vertex directly), but it isn't necessary because we can use the equation given to us. Though I recommend using a graphing calculator to confirm the answer.

The original function is the same as y = 1x^2+8x + (-2). We see that it is in the form y = ax^2+bx+c where

a = 1

b = 8

c = -2

Use the values of 'a' and b to get the value of h, which is the x coordinate of the vertex.

h = -b/(2a)

h = -8/(2*1)

h = -4

The x coordinate of the vertex is x = -4. Plug this into the original equation to get

f(x) = x^2+8x-2

f(-4) = (-4)^2 + 8(-4) - 2

f(-4) = -18

Plugging x = -4 into f(x) leads to y = -18. The point (-4, -18) is on the parabola. Furthermore, this is the vertex (h,k)

------------

Alternatively, you can complete the square as shown below

y = x^2 + 8x - 2

y = (x^2 + 8x) - 2

y = (x^2 + 8x + 0) - 2

y = (x^2 + 8x + 16 - 16) - 2

y = (x^2 + 8x + 16) - 16 - 2

y = (x+4)^2 - 18

y = 1(x+4)^2 - 18

y = 1(x-(-4))^2 - 18

The last equation is in the form y = a(x-h)^2 + k with (h,k) = (-4,-18) being the vertex. The 16 is the result of taking half of 8 and squaring that result. We have 16-16 = 0 to make sure that we don't change the equation and keep things balanced. This is the same as adding 16 to both sides. All of this is done so we can end up with the (x+4)^2 perfect square portion.You can expand out (x+4)^2 - 18 and you should get x^2+8x-2 again.

D is the answer

Because you need to rewrite in vertex form and use this form to find the vertex (h,k)

Which should get you to

(-4,-18)

find the value of a answers: a:15 b:14 c:19 d:16

Answers

Answer:

6a+10= 3a+55

6a-3a= 55-10

3a= 45

a= 45/3

a= 15

The answer is (A) 15

Hope this helps^°^

Answer:

A. 15

Step-by-step explanation:

The angles are vertical angles, therefore they are congruent. We can set the two measures equal to each other.

[tex]6a+10=3a+55[/tex]

Now, solve for a. We must get a by itself on one side of the equation.

First, subtract 3a from both sides of the equation.

[tex]6a-3a+10=3a-3a+55[/tex]

[tex](6a-3a)+10=55[/tex]

[tex]3a+10=55[/tex]

Next, subtraction 10 to both sides of the equation. We subtract because 10 is being added to 3a. The inverse of addition is subtraction.

[tex]3a+10-10=55-10[/tex]

[tex]3a=55-10[/tex]

[tex]3a=45[/tex]

3 and a are being multiplied. The inverse of multiplication is division. Divide both sides of the equation by 3

[tex]3a/3=45/3[/tex]

[tex]a=45/3[/tex]

[tex]a=15[/tex]

The value of a is 15 and choice A is correct.

TELL ME THE FORMULA FOR VOLUME AND SURFACE AREA OF PRISMS

Answers

Answer:

surfacevolume = 1/2 (bh) l

surfacearea = bh + pl

So the answer is bh plus pl and I hopes this help good luck
Other Questions
Which term describes a time period marked by a change that begins a new period of development? century decade era millennium Which property is shown by -6 + 0 = -6? A. Commutative Property of Addition B. Identity Property of Addition C. Distributive Property D. Associative Property of Addition can someone help on this question mr.wright judges the annual jelly bean challenge at the summer fair.every year he encourages the citizens in his town to guess the number of jelly beans in the jar.he keeps in record of everyones guesses and the number of the jelly beans each person was off by. what is the independent and dependent quantity? An interference pattern is produced by light with a wavelength 590 nm from a distant source incident on two identical parallel slits separated by a distance (between centers) of 0.580 mm .Required:a. If the slits are very narrow, what would be the angular position of the first-order, two-slit, interference maxima?b. What would be the angular position of the second-order, two-slit, interference maxima in this case? How do I know if CsBr in water is an acid, base or neutral? And how do I show it with a balanced equation Uue5) Identify each boldfaced prepositional phrase as either an adjectival phrase or an adverbialphrase.The sign cautioned swimmers to stay within the designated swimmingarea.The lifeguards on the beach watched the swimmers carefully.Children and their parents built sandcastles with tall towers.Seagulls soared over the water, searching for food.6) Select the Which phrase describes a feature of a constitutional liberal democracy? A- closed-session voting without journalists present B- a highly centralized government administration C- leaders who are military officials or religious clerics D- a system of laws that applies to leaders and citizens equally Under Roosevelts leadership, the United States sent troops to occupy? Which property justifies the following equation? 7[6+5+(-6)] = [6+(-6)+5] A.distributive B.commutative C.associative D.identity Pamela drove her car 99 kilometers and used 9 liters of fuel. She wants to know how many kilometers (k)left parenthesis, k, right parenthesis she can drive with 12 liters of fuel. She assumes the relationship between kilometers and fuel is proportional.How many kilometers can Pamela drive with 12 liters of fuel? By the end of the Ming Dynasty, how many ports did they have open for trade?A. 1,500B. 3,000C. 500D. 3 a number has 2,5 and 7 as its prime factors. what are the four smallest values it and take What is 25x + 67y if x = 23 and y = 36. Give explanation please! Can someone help?????????? Please Help Asap will give brainliest!!! M(9, 8) is the midpoint of side RS.The coordinates of S are (10, 10). What are the coordinates of R? No nonsense answers will report and give explanation plz. who is the father of Genetic help please! Darren is finding the equation in the form y = m x + b for a trend line that passes through the points (2, 18) and (3, 8). Which value should he use as b in his equation? a) 34 b) 19 c) 2 d) 14 Fill in the blank:Juan experienced a _____ memory during therapy for depression. This type of memory is a memory that is formed, but never really happened. During his depression, he remembered many sad times in his life because of his mood and emotions. Juan experienced_____ -dependent recall Which of the following functions has a vertical asymptote at x = 2, a horizontalasymptote at f(x) = 1, and a root at x = -1?A.f(x) = 2 + 1B.f(x) = x 2 + 1c.f(x) = x 2 - 1D.f(x) == +1